Mathematical Systems - Modular Arithmetic, Applications of Modular Arithmetic, Introduction to Group PDF

Title Mathematical Systems - Modular Arithmetic, Applications of Modular Arithmetic, Introduction to Group
Author Shannie Fe Perez Bagasina
Course BS in Management Accounting
Institution University of Makati
Pages 11
File Size 166.5 KB
File Type PDF
Total Downloads 119
Total Views 177

Summary

Modular Arithmetic, Applications of Modular Arithmetic, Introduction to Group Theory...


Description

MODULE 10 MATHEMATICAL SYSTEMS (Modular Arithmetic, Applications of Modular Arithmetic, Introduction to Group Theory)

III. ACTIVITIES TOPIC 1 Modular Arithmetic 1. Calculate the following using a 12-hour clock a. 5

8 = 1

b. 10

11 = 9

c.

6= 4

10

d. 1

4= 9

2. Determine whether the congruence is true or false. Justify your answer. a. 5 ≡ 20 mod 4 false not a congruence because (5 – 20)/4 does not divide or when you divided 5 by 4, the quotient will be 1 remainder 1 not 20. b. 100 ≡ 4 mod 8 is true congruence because (100 – 4)/8 does divide or when you divided 100 by 8, the quotient will be 12 remainder 4, that means the congruence is true. c. 25 ≡ 85 mod 12 is true congruence because (25 – 85)/12 = -5 does divide. d. 7 ≡ 21 mod 3 false not a congruence because (7 – 21)/3 does not divide or when you divided 7 by 3, the quotient will be 2 remainder 1 not 21. 3. Perform the modular arithmetic a. (42 + 35) mod 3 Answer: 42 + 35 = 77 divided by 3 = 25 remainder 2, therefore the answer is (42 + 35) mod 3 = 2 b. (19 – 6) mod 5 Answer: 19 – 6 = 13 divided by 5 = 2 remainder 3, therefore the answer is (19 – 6) mod 5 = 3 https://docs.google.com/forms/u/0/d/e/1FAIpQLSfcjKQ_Je5W2NQs5XEaEx8QaU3xfrBvUe f2iTkd4AP1v7muIw/formResponse https://docs.google.com/forms/d/e/1FAIpQLSfcjKQ_Je5W2NQs5XEaEx8QaU3xfrBvUef2iT kd4AP1v7muIw/viewscore? viewscore=AE0zAgAoHiX6XMwskrsxFX3eKv17HgkfBCmD3blH1A9PZsHBf8wc5I0tD0pFwN0AA c. (5 • 12) mod 4 Answer: 5 x 12 = 60 divided by 4 = 15, therefore the answer is (5 • 12) mod 4 = 0

d. (26 • 11) mod 15 Answer:

26 x 11 = 286 divided by 15 = 19 remainder 1, therefore the answer is (26 • 11) mod 15 = 1

4. Find all whole number solutions of the congruence equation. a. (5x + 4) ≡ 2 mod 8 Solution: x=0

5(0) + 4 ≢ 2 mod 8

not a solution

x=1

5(1) + 4 ≢ 2 mod 8

not a solution

x=2

5(2) + 4 ≢ 2 mod 8

not a solution

x=3

5(3) + 4 ≢ 2 mod 8

not a solution

x=4

5(4) + 4 ≢ 2 mod 8

not a solution

x=5

5(5) + 4 ≢ 2 mod 8

not a solution

x=6

5(6) + 4 ≡ 2 mod 8

A solution

x=7

5(7) + 4 ≢ 2 mod 8

not a solution

x=8

5(8) + 4 ≢ 2 mod 8

not a solution

x=9

5(9) + 4 ≢ 2 mod 8

not a solution

The solution between 0 and 9 is 6. b. (3x + 12) ≡ 7 mod 10 Solution: x=0

3(0) + 12 ≢ 7 mod 10

not a solution

x=1

3(1) + 12 ≢ 7 mod 10

not a solution

x =2

3(2) + 12 ≢ 7 mod 10

not a solution

x=3

3(3) + 12 ≢ 7 mod 10

not a solution

x=4

3(4) + 12 ≢ 7 mod 10

not a solution

x=5

3(5) + 12 ≡ 7 mod 10

A solution

x=6

3(6) + 12 ≢ 7 mod 10

not a solution

x=7

3(7) + 12 ≢ 7 mod 10

not a solution

x=8

3(8) + 12 ≢ 7 mod 10

not a solution

x=9

3(9) + 12 ≢ 7 mod 10

not a solution

The solution between 0 and 9 is 5.

TOPIC 2 Group Theory 5. Does the set { , 1, 2, 3, 4} with operation multiplication module 5 form a group? TOPIC 3 Arithmetical Operations with Complex Numbers 6. Perform as indicated in each item: a. ADD: (12 + 8i) + (6 – 4i) Answer: (12 + 8i) + (6 – 4i) = (12 + 6) + (8 – 4)i = 18 + 4i

Thus, (12 + 8i) + (6 – 4i) = 18 + 4i b. ADD: (6 + √-144) + (3 − √-81) Answer; Step 1 (6 + √-144) = (6 + 12i) (3 − √-81) = (3 − 9i) Step 2 (6 + 12i) + (3 − 9i) = (6 + 3) + (12i − 9i) = 9 + 3i Thus, (6 + √-144) + (3 − √-81) = 9 + 3i c. SUBTRACT: (25 + 18i) – (14 – 6i) Answer: 25 + 18i − 14 + 6i = (25 – 14) + (18i + 6i) = 11 + 24i Thus, (25 + 18i) – (14 – 6i) = 11 + 24i d. SUBTRACT: (22 − √-49) – (12 − √-16) Answer: Step 1 (22 − √-49) = (22 – 7i) (12 − √-16) = (12 – 4i) Step 2 22 – 7i − (12 – 4i) = 22 – 7i – 12 + 4i = (22 – 12) + (-7i + 4i) = 10 – 3i Thus, (22 − √-49) – (12 − √-16) = 10 – 3i e. MULTIPLY: (8 + 5i) • (6 + 2i) Answer: 8(6 + 2i) + 5i(6 + 2i) = (8 x 6) + (8 x 2i) + (5i x 6) + (5i x 2i) = 48 + 16i + 30i + 10i² = -1 then, = 48 + 16i + 30i + 10(-1) = 48 + 46i – 10 = 38 + 46i Thus, (8 + 5i) • (6 + 2i) = 38 + 46i

f.

MULTIPLY: (6 + √-25) • (2 − √-121) Answer: Step 1 (6 + √-25) = (6 + 5i) (2 − √-121) = (2 – 11i) Step 2 (6 + 5i) • (2 – 11i) = 6(2 – 11i) + 5i(2 – 11i) = (6 x 2) − (6 x 11i) + [(5i x 2) – (5i x 11i)] = 12 – 66i + 10i + 55i² = 12 – 66i + 10i + 55(-1) = 12 – 56i – 55

= −43 – 56i Thus, (6 + √-25) • (2 − √-121) = −43 – 56i TOPIC 4 Cryptology 7. Use a cyclical alphabetic encrypting code that shifts the letters stated number of positions to decode the encrypted message for 8th positions: VWJWLG QA XMZNMKB. Answer: c ≡ (p + 8) mod 26 V : c ≡ (22 + 8) mod 26 = 30 mod 26 = 4. So V will be coded by D W : c ≡ (23 + 8) mod 26 = 31 mod 26 = 5. So W will be E J : c ≡ (10 + 8) mod 26 = 18 mod 26. So J will be R W : c ≡ (23 + 8) mod 26 = 31 mod 26 = 5. So W will be E L : c ≡ (12 + 8) mod 26 = 20 mod 26. So L will be T G : c ≡ (7 + 8) mod 26 = 13 mod 26. So G will be O Therefore, VWJWLG is decoded as DERETO Q : c ≡ (17 + 8) mod 26 = 25 mod 26. So Q will be Y A : c ≡ (1 + 8) mod 26 = 9 mod 26. So A will be I Therefore, QA is decoded as YI X : c ≡ (24 + 8) mod 26 = 32 mod 26 = 6. So X will be F M : c ≡ (13 + 8) mod 26 = 21 mod 26. So M will be U Z : c ≡ (0 + 8) mod 26 = 8 mod 26. So Z will be H N : c ≡ (14 + 8) mod 26 = 22 mod 26. So N will be V M : c ≡ (13 + 8) mod 26 = 21 mod 26. So M will be U K : c ≡ (11 + 8) mod 26 = 19 mod 26. So K will be S B : c ≡ (2 + 8) mod 26 = 10 mod 26. So B will be J. Therefore, XMZNMKB is decoded as FUHVUSJ

8. Use a cyclical alphabetic encrypting code to decode the encrypted message: AOB HVS HCFDSRCSG

9. Use the encrypting congruence c ≡ (3p + 2) mod 26 to code the message TOWER OF LONDON. Answer: c ≡ (3p + 2) T : c ≡ 3(20) + 2 = 62 mod 26 = 2 remainder 10. So T will be coded by J O : c ≡ 3(15) + 2 = 47 mod 26 = 21. So O will be U W : c ≡ 3(23) + 2 = 71 mod 26 = 2 remainder 19. So W will be S E : c ≡ 3(5) + 2 = 17 mod 26. So E will be Q R : c ≡ 3(18) + 2 = 56 mod 26 = 2 remainder 4. So R will be D Therefore, TOWER is coded as JUSQD O : c ≡ 3(15) + 2 = 47 mod 26 = 21. So O will be U F : c ≡ 3(6) + 2 = 20 mod 26. So F will be T Therefore, OF is coded as UT

L : c ≡ 3(12) + 2 = 38 mod 26 = 12. So L will be L O : c ≡ 3(15) + 2 = 47 mod 26 = 21. So O will be U N : c ≡ 3(14) + 2 = 44 mod 26 = 18. So N will be R D : c ≡ 3(4) + 2 = 14 mod 26 . So D will be N O : c ≡ 3(15) + 2 = 47 mod 26 = 21. So O will be U N : c ≡ 3(14) + 2 = 44 mod 26 = 18. So N will be R Therefore, LONDON is coded as LURNUR

10. Decode the message LOFT JGMK LBS MNWMK that was encrypted using the congruence c ≡ (3p + 4) mod 26 Answer: c ≡ (3p + 4) mod 26 L : c ≡ 3(12) + 4 = 40 mod 26 = 14. So, L will be N O : c ≡ 3(15) + 4 = 49 mod 26 = 23. So, O will be W F : c ≡ 3(6) + 4 = 22 mod 26. So, F will be V T : c ≡ 3(20) + 4 = 64 mod 26 = 2 remainder 12. So, T will be L Therefore, LOFT is decoded as NWVL J : c ≡ 3(10) + 4 = 34 mod 26 = 8. So, J will be H G : c ≡ 3(7) + 4 = 25 mod 26. So, G will be Y M : c ≡ 3(13) + 4 = 43 mod 26 = 17. So, M will be Q K : c ≡ 3(11) + 4 = 37 mod 26 = 11. So, K will be K Therefore, JGMK is decoded as HYQK L : c ≡ 3(12) + 4 = 40 mod 26 = 14. So, L will be N B : c ≡ 3(2) + 4 =10 mod 26. So, B will be J S : c ≡ 3(19) + 4 = 61 mod 26 = 2 remainder 9. So, S will be I Therefore, LBS is decoded as NJI M : c ≡ 3(13) + 4 = 43 mod 26 = 17. So, M will be Q N : c ≡ 3(14) + 4 = 46 mod 26 = 20. So, N will be T W : c ≡ 3(23) + 4 =73 mod 26 = 2 remainder 21. So, W will be U M : c ≡ 3(13) + 4 = 43 mod 26 = 17. So, M will be Q K : c ≡ 3(11) + 4 = 37 mod 26 = 11. So, K will be K Therefore, MNWMK is decoded as QTUQK

IV. ASSESSMENT TOPIC 1 Modular Arithmetic 1. Calculate the following using a 12-hour clock a. 11

5=4

b. 5

11 = 6

c. 8

8= 4

d. 5

5 = 12

2. Determine whether the congruence is true or false. Justify your answer. a. 21 ≡ 45 mod 6 is true congruence because (21 – 45)/6 is -4 does divide. e. 5 ≡ 20 mod 4 false not a congruence because (5 – 20)/4 does not divide or when you divided 5 by 4, the quotient will be 1 remainder 1 not 20. b. 72 ≡ 30 mod 5 false not a congruence because (72 – 30)/5 does not divide or when you divided 72 by 5, the quotient will be 8 remainder 32 not 30. c. 18 ≡ 60 mod 7 is true congruence because (18 – 60)/7 is -6 does divide.

3. Perform the modular arithmetic a. (9 + 15) mod 7 Answer: 9 + 15 = 24 divided by 7 = 3 remainder 3, therefore the answer is (9 + 15) mod 7 = 3 b. (8 – 15) mod 12 Answer: 8 – 15 = -7 divided by 12 = does not divide c. (6 • 8) mod 9 Answer: 6 x 8 = 48 divided by 9 = 5 remainder 3, therefore the answer is (6 • 8) mod 9 = 3 d. (14 • 20) mod 8 Answer: 14 x 20 = 280 divided by 8 = 35, therefore the answer is (14 • 20) mod 8 = 0

4. Find all whole number solutions of the congruence equation. a. (3x + 12) ≡ 7 mod 10 Solution: x=0 x=1 x=2 x=3 x=4 x=5 x=6 x=7 x=8 x=9

3(0) + 12 ≢ 7 mod 10 3(1) + 12 ≢ 7 mod 10 3(2) + 12 ≢ 7 mod 10 3(3) + 12 ≢ 7 mod 10 3(4) + 12 ≢ 7 mod 10 3(5) + 12 ≡ 7 mod 10 3(6) + 12 ≢ 7 mod 10 3(7) + 12 ≢ 7 mod 10 3(8) + 12 ≢ 7 mod 10 3(9) + 12 ≢ 7 mod 10

The solution between 0 and 9 is 5. b. 3x ≡ 8 mod 11 Solution:

not a solution not a solution not a solution not a solution not a solution A solution not a solution not a solution not a solution not a solution...


Similar Free PDFs